1
$\begingroup$

Let $K$ be a local field, with a completed algebraic closure $\hat{\bar{K}}$ and $A$ be an affinoid $K$-algebra. Is it then true, that for $f_0, \ldots ,f_n \in A$, the map $\phi:$ Sp$A \rightarrow$ $\mathbb{P}_K^{n,rig}, x \mapsto [f_0(x):\ldots:f_n(x)]$ defines a morphism of rigid spaces?

I know by a lecture a similar result for varieties, but the proof I have is not applicable in this case.

EDIT: To clarify what it means. For $f \in A$ and $x \in$ Max$A$, $f(x)$ is the residue class in $A/x$, which can be embedded in $\bar{K}$, so $f(x) \in \bar{K}$, is determined up to conjugation. Points in $\mathbb{P}_K^{n,rig}$ are also meant as equivalence classes of conjugated elements of $\mathbb{P}^{n,rig}(\hat{\bar{K}})$ (see https://ivv5hpp.uni-muenster.de/u/pschnei/publ/pap/xsymm.pdf, page 8)

I thought what you need is

  1. $\phi$ is continuous with respect to the Grothendieck topology of rigid spaces
  2. for all admissible open $U \subset \mathbb{P}_K^{n,rig}$ and for all $f \in \mathcal{O}_{\mathbb{P}_K^{n,rig}}(U)$, we have that $f\circ \phi_{|\phi^{-1}(U)} \in \mathcal{O}_{\text{Sp}A}(\phi^{-1}(U))$, i.e we have a map $\phi^*_U:\mathcal{O}_{\mathbb{P}_K^{n,rig}}(U) \rightarrow \mathcal{O}_{\text{Sp}A}(\phi^{-1}(U))$ compatible with restrictions

As rigid geometry is relatively new for me, both points, if true, are at the moment not obvious for me and I don't know what a good way could be to verify this.

$\endgroup$
2
  • $\begingroup$ Let $R>0$ that is bigger than all the norms on the $f_i$'s on $\mathrm{Sp}(A)$. Then, you have a bounded morphism from the Tate algebra $K \langle R^{-1} \underline{T} \rangle$ to $A$, hence a morphism from $\mathrm{Sp}(A)$ to a disc of dimension $n+1$. You can embed the latter into $\mathbb{A}^{n+1,\mathrm{rig}}_K$ and then project onto $\mathbb{P}^{n,\mathrm{rig}}_K$. Is this want you wanted? $\endgroup$ Sep 27, 2019 at 8:45
  • $\begingroup$ Mmmh not sure. I got it from ivv5hpp.uni-muenster.de/u/pschnei/publ/pap/xsymm.pdf, page 10. I added some lines to hopefully clarify the situation. $\endgroup$
    – KKD
    Sep 27, 2019 at 10:11

0

Your Answer

By clicking “Post Your Answer”, you agree to our terms of service and acknowledge you have read our privacy policy.

Browse other questions tagged or ask your own question.